a circuit maintains a constant resistance. if the current in the circuit is doubled, what is the effect on the power dissipated by the circuit? a circuit maintains a constant resistance. if the current in the circuit is doubled, what is the effect on the power dissipated by the circuit? the power dissipated is reduced by a factor of 2. the power dissipated is doubled. the power dissipated is reduced by a factor of 4. the power dissipated remains constant. the power dissipated is quadrupled.

Answers

Answer 1

When the circuit's current doubles, the power lost in the circuit is given by, Beginning with "begin aligned" and ending with "end aligned," P =I 0 2 R =(2 I) 2 R =4 I 2 R. The power dissipated by the circuit is quadrupled if the current in the circuit is twice.

The idea of a resistance's power loss at constant voltage. The square of the potential across the resistor over its resistance, then, represents the power dissipated or the bubble that generated at constant volume at constant voltage. From this, we might conclude that power and resistance are inversely related.

Starting with "begin aligned" and continuing with "end aligned," P =I 0 2 R =(2 I) 2 R =4 I 2 R gives the power lost in the circuit when the circuit's current doubles. If the circuit's current is doubled, the power it dissipates is multiplied by four.

Now that we know this, we may write the original power P equals are over our buddies and the power BI prime over it. We can express the power P prime equals P times are greater than our prime from this. The resistance is prime is off by double the initial help in the problem now.

As a result, the power Well will decrease by a factor of 2, and the new power P prime will therefore be P over two.

To know more about circuit click on the link:

https://brainly.com/question/12608516

#SPJ4


Related Questions

Your community is considering building a dam on a nearby river to reduce flooding. Would you support this proposal? Explain.
I need to get it done right now

Answers

Your community is considering building a dam on a nearby river to reduce flooding. yes, this proposal need support. because through dam we can store flood water.

What is a dam?

A physical barrier known as a dam slows or limits the flow of surface or subsurface water. Reservoirs built by dams provide water for navigation, aquaculture, irrigation, human consumption, industrial use, and other purposes.

What is the use of dam?

Let's examine some of a dam's uses now that we have a basic understanding of what a dam is. It is valued as a resource for water. They provide water for a variety of uses, including home, agricultural, and industrial ones. Dams are also utilised to produce energy and improve river transportation. Dams are used more and more frequently in daily life. Dams also promote local tourism, which contributes to the growth of the nation. Dams can be used for many different things.

Learn more about dam here :

brainly.com/question/29315

#SPJ13

what factors affect skill-related fitness and how does each affect it?

Answers

Several factors affect your skill-related fitness and your skills, including heredity, practice etc .

Skill-related fitness is have six different components; agility, speed, power, balance, coordination, reaction time.

Several factors affect your skill-related fitness and your skills, including heredity, practice etc

for example some people are able to run faster than others or react quick because they inherited these traits from their parents. A person who did not inherited a tendency in these areas from their ancestors ,may have more difficulty performing skills that require those abilities.

To learn more about skill related fitness :

https://brainly.com/question/1318571

#SPJ1

17. A ball rolls off a lab table at 5 m/s horizontally. The table is 1.2 meters tall.

a. How long is it in the air?

b. How far from the table does it hit the ground?

c. What are the x and y speeds just before it hits the ground?

Answers

The ball is in the air for 0.24 seconds, the ball hits the ground 2.46 meters far from the table and the x speed is 0m/s and y speed is 5m/s.

Velocity of the ball is 5m/s and the height of the table is 1.2 meters.

a. We know,

Velocity = distance/time

5 = 1.2/time

Time = 1.2/5

Time = 0.24 seconds.

The time taken by the ball to reach the ground is 0.24 seconds.

b. The ball after rolling of the table will behave like Horizontal projectile.

Horizontal range R formula,

R = V.√(2h/g)

Putting all the values,

R = 5.√(2×1.2/9.8)

R = 5×0.49

R = 2.46 meters.

c. When the ball reaches the ground,

X speed is 0m/s and Y speed is 5m/s.

To know more about horizontal projectile, visit,

https://brainly.com/question/19443942

#SPJ1

consider a resistor-inductor circuit where a 10v force is applied for a period of 2 seconds. the equation for the current in the circuit is given: \f$\displaystyle \small \frac{di}{dt} 5i

Answers

A step function is defined U(t)=0,1 [tex]\left \{ {{t < 0} \atop {t > 0}} \right.[/tex]  i(t)=e^-5t + 2 if t=2&3 & i(t)=3e^-5t if  2>t>3.

A step function, sometimes known as a staircase function, is a piecewise constant function with a limited number of parts, according to the definition given in mathematics. In other words, one can think of a function on the real numbers as a finite linear combination of indicator functions of given intervals. In addition, it goes by the names floor function and biggest integer function. There is no continuity in the step function. In mathematics, a step function is a function whose graph resembles a series of steps because it is made up of numerous horizontal line segments interspersed with jumps. The biggest integer function is a frequent illustration of step functions in mathematics.

When provided intervals are used, a step function has a constant value, but the constant varies for each interval. A function that returns the biggest integer that is less than or equal to x is known as the greatest integer function.

Learn more about Step function here:

https://brainly.com/question/12701460

#SPJ4

a cylinder of mass m slides down from rest in a vertical tube whose inner surface is covered by a viscous oil of film thickness h. if the diameter and height of the cylinder are d and l, respectively, derive an expression for the velocity of the cylinder as a function of time, t. discuss what will happen as t → [infinity]. can this device serve as a viscometer?

Answers

When there is relative motion between the fluid layers or between the fluid and solid surface, the fluid's internal property known as viscosity is created. This relative motion is being resisted by viscosity.

Derivations of the equations of motion—the connections for the conservation of mass and momentum—in rectangular,

coordinates that are spherical and cylindrical. The entire body of experimental data suggests that the fundamental equations of fluid mechanics are in fact those listed below, and that they apply to any circumstance where a Newtonian fluid is flowing, in theory.

Viscosity, a characteristic of fluids, is produced when there is relative motion between the fluid layers or between the fluid and solid surface. Viscosity prevents this relative motion from happening.

Unfortunately, due to their all-encompassing nature, their analytical solution is challenging or impossible unless the circumstances are quite straightforward.

However,

These "Navier-Stokes equations" must be understood for the following reasons.

reasons:

1. They result in the analytical and precise resolution of some basic yet significant problems.

issues.

2. They serve as the foundation for additional chemical engineering research in various fields.

3. A few reasonable simplification assumptions can frequently result in

for many engineering problems, approximation solutions are eminently acceptable purposes. Typical examples can be found in the study of boundary layers, the use of films to cover substrates, waves, lubrication, and inviscid (irrotational) flow.

To know more about viscosity click on the link:

https://brainly.com/question/2193315

#SPJ4

an initially stationary box of sand is to be pulled across a floor by means of a cable in which the tension should not exceed 1100 n. the coefficient of static friction between the box and the floor is 0.35. (a) what should be the angle between the cable and the horizontal in order to pull the greatest possible amount of sand, and (b) what is the weight of the sand and box in that situation?

Answers

a) The angle between the cable and the horizontal in order to pull the greatest possible amount of sand is 19°. b) The weight of the sand and box in that situation is 3.3x10³ N.

A similar scenario is depicted with a free-body diagram and in the textbook's illustration, where the unknown angle is represented by the symbol. We employ the same system of coordinates as in that figure.

a) Newton's second law thus results in

x : T cosФ - f = ma

y : T sinФ + Fn-mg = 0

With a = 0 and f = f s, max = s F N, we may solve for the mass of the box-and-sand (as a function of angle) as follows:

m = T/g (sinФm + cosФm/us )

Hence, in order to determine the angle m that corresponds to the greatest mass that may be drawn, we will solve using calculus techniques.

dm/dt = T/g  (sinФm + cosФm/us ) = 0

As a result, tan m = s is obtained, and for s = 0.35, m = 19 is obtained.

(b) The result of entering our value for m into the formula we discovered for the mass of the box and sand is m=340 kg. This is equivalent to a weight of mg=3.3x10³ N.

To know more about angle click on the link:

https://brainly.com/question/28451077

#SPJ4

Is the exoplanet like earth in terms of its distance from its star? Explain your answer

Answers

Every planet in the solar system orbits the Sun. Planets known as exoplanets orbit other stars. Astronomers search for exoplanets by observing the effects that these planets have on the stars they orbit.

What is the exoplanet?

Any planet outside of our solar system is an exoplanet. The majority of exoplanets orbit other stars, while rogue planets—free-floating exoplanets that are unattached to any star—orbit the galactic center.

Does the exoplanet have life?

Since liquid water is necessary for all known forms of life, an exoplanet that is too near or too far from its host star is less likely to support life. Additionally, some primordial compounds may require ultraviolet light to form, but a hyperactive host star can blast the nascent planet, removing its atmosphere.

To know more about Exoplanet visit:

https://brainly.com/question/9991501

#SPJ13

an airplane traveling 1001 m above the ocean at 125 km/h is going to drop a box of supplies to shipwrecked victims below. a. how many seconds before the plane is directly overhead should the box be dropped?

Answers

14.3 seconds before the plane is directly overhead should the box be dropped in projectile motion.

The motion of thebox is a projectile motion, therefore it consists of two independent motions:  

A uniform motion (at constant velocity) along the horizontal direction  

A uniformly accelerated motion, with constant acceleration (=acceleration of gravity, ) in the downward direction  

s = 1000 m is the vertical distance that the box must cover

u = 0 (the box's initial vertical velocity (zero, since it is travelling horizontally)

g is the acceleration of gravity

t is the time

The time the box needs to arrive at the ground is determined by solving for t:

[tex]t=\sqrt((2s/g=\sqrt((2(1000))/(9.8))=14.3s[/tex]

To know more about  projectile motion, visit : https://brainly.com/question/11049671

#SPJ4

A cougar with a mass of 80 kg is standing at the edge of a cliff. Find the cougar's potential energy if the cliff is 70 m high. show your work !!!

Answers

The potential energy of the 80kg cougar at a height of 70m is 54880 Joules.

What is the potential energy of the cougar?

Potential energy is simply the energy a body possesses due to its position.

It is expressed mathematically as;

P.E = mgh

Where m is mass, h is height and g is acceleration due to gravity ( g = 9.8m/s² )

Given the data in the question;

Mass of the cougar m = 80kgHeight h = 70mPotential energy P.E = ?

Plug the given values into the potential energy formula above and solve.

P.E = mgh

P.E = 80kg × 9.8m/s² × 70m

P.E = 54880kgm²/s²

P.E = 54880 Joules.

Therefore, the potential energy of the cougar is 54880 Joules.

Learn more about potential energy here: https://brainly.com/question/11749818

#SPJ1

A scientific model can be used to

Answers

Scientific models area unit accustomed make a case for and predict the behaviour of real objects or systems.

A system of ideas, events, or processes is described physically, mathematically, conceptually, or conceptually in an exceedingly scientific model. By victimisation their scientific experience to produce explanations that create it doable to forecast the patterns, scientists work to recognise and comprehend the patterns that exist in the environment.

Scientific model illustrations include:

a simulation of the sun, moon, and earth's motions (which you participated in last year) a way for statement eclipses. The weather will be foretold victimisation models that describe weather events.

Models will assist you in mentally imagining one thing that's difficult to ascertain or comprehend. Models will aid in scientific prediction, understanding of processes, and communication of ideas.

To learn more about scientific models click at https://brainly.com/question/1462653

gas giants have many moons than terristcal planets due to having higher ___ ?

Answers

Gas giants have more moons than terrestrial planets due to having higher masses.

What are giant planets?

Giant planets are composed of low boiling point substances rather than other solid substances but massive solid planets can also exist. There are four giant planets in the Solar System i.e. Jupiter, Saturn, Uranus, and Neptune. They are composed of hydrogen and helium atom i.e. Jupiter and Saturn whereas Uranus and Neptune are composed of ice, rock, hydrogen, and helium.

The giant planets have more moons as compared to the terrestrial planets because of their big masses which leads to stronger gravitational fields. Giant planets also occupy greater space and have larger volumes of mass in their surrounding atmosphere.

So we can conclude that due to having higher masses, Gas giants have many moons as compared to terrestrial planets.

Learn more about planets here: https://brainly.com/question/13106554

#SPJ1

A 11.0kg ball traveling at 13.0m/s is crashed into from behind by a 14.0kg ball traveling at 22.0m/s in the same direction. What is the velocity of the first ball if the second slows down to 16.2m/s after the collision?

Answers

Let p be the total linear momentum of the system before the collision and p' the total linear momentum of the system after the collision. Let p₁₁ and p₁₄ be the linear momentum of the 11.0kg ball and the 14.0kg ball before the collision, and let p₁₁' and p₁₄' be their linear momenta after the collision.

According to the Law of Conservation of Linear Momentum, the total linear momentum of the system before and after the collision remains the same. Then:

[tex]\begin{gathered} p=p^{\prime} \\ \Rightarrow p_{11}+p_{14}=p_{11}^{\prime}+p_{14}^{\prime} \end{gathered}[/tex]

Let v be the unknown speed of the 11.0kg ball after the collision. Find the value of p₁₁, p₁₄, and p₁₄'. Find an expression for p₁₁' in terms of v and replace all the values into the above equation. Solve for v.

The linear momentum of a particle with mass m and velocity v is:

[tex]p=mv[/tex]

Then:

[tex]\begin{gathered} p_{11}=(11kg)(13.0\frac{m}{s})=143kg\cdot\frac{m}{s} \\ \\ p_{14}=(14kg)(22.0\frac{m}{s})=308kg\cdot\frac{m}{s} \\ \\ p^{\prime}_{11}=(11kg)\cdot v \\ \\ p^{\prime}_{14}=(14kg)(16.2\frac{m}{s})=226.8kg\cdot\frac{m}{s} \end{gathered}[/tex]

Then:

[tex]\begin{gathered} p_{11}+p_{14}=p_{11}^{\prime}+p_{14}^{\prime} \\ \\ \Rightarrow143kg\cdot\frac{m}{s}+308kg\cdot\frac{m}{s}=(11kg)\cdot v+226.8kg\cdot\frac{m}{s} \\ \\ \Rightarrow143kg\cdot\frac{m}{s}+308kg\cdot\frac{m}{s}-226.8kg\cdot\frac{m}{s}=(11kg)\cdot v \\ \\ \Rightarrow224.2\cdot\frac{m}{s}=(11kg)\cdot v \\ \\ \Rightarrow v=\frac{224.2kg\cdot\frac{m}{s}}{11kg}=20.381818\ldots\frac{m}{s} \\ \\ \therefore v\approx20.4\frac{m}{s} \end{gathered}[/tex]

Therefore, the velocity of the first ball after the collision is 20.4m/s.

electromagnetic radiation with a maximum wavelength of 540 nm(5.4×10−7 m) is needed for the study of the photoelectric effect in potassium atoms. what is the approximate frequency that corresponds to this wavelength?

Answers

At a speed of two times ten to the eighth and violent. 5, 40 times 10 to the power -9 meters is written as. This leads to the conclusion that frequency F is equivalent to 5.6 times. Turn 14 times every second. Therefore, the given electromagnetic radiation requires this frequency.

In this problem, we have a given violent electromagnetic radiation, and we need to determine the frequency that corresponds to that violent electromagnetic radiation.

The chosen wavelength is lambda. I called at five minutes and forty nanometers—540 times 10 to the power—or nine meters. Considering that one nanometer equals 10 to the power -9 meters. Now, sea upon lambda finds frequency.

The value of C is three in 2, ten to the power eight meters per second, violent, and the speed of light in a vacuum. 5, 40 times 10 to the power -9 meters is written as. This leads to the conclusion that frequency F is equivalent to 5.6 times. Turn 14 times every second. Therefore, the given electromagnetic radiation requires this frequency.

To know more about electromagnetic radiation click on the link:

https://brainly.com/question/10759891

#SPJ4

What type of wave is an electromagnetic wave?
transverse
longitudinal
both
neither

Answers

Both.
Electromagnetic waves could be transverse or longitude or a combination of both.

X 24. A man drives a car and records the distance he travels on a distance- *0/2
time graph. At one point he gets stuck in traffic and does not move for
one hour. This point on the graph the slope will be horizontal. Why is this correct?

Answers

The distance-time graph will show the slope as horizontal for the time in which the man don't move.

How to find slope of a graph?

The slope formula rise/run is used to get the slope from a graph. The first thing we must do when given a line's graph and asked to determine its equation is to determine its slope. In order to determine the slope from a graph, we can take any two points on the line (x1, y1) and (x2, y2) and use the formula (y2 - y1) / (x2 - x1).

In the distance-time graph the distance is represented at y-axis and the time is represented at x-axis. So, In the time in which the man stops moving (stuck in the traffic) there is no change in distance but the times keeps changing or increasing that's why at the point he gets stuck in traffic and does not move for one hour. This point on the graph the slope will be horizontal.

To know more about slope of a graph, go to link

https://brainly.com/question/19376563

#SPJ9

Find the x -component of vector a⃗ = (7.0 m/s2 , −y -direction).

Answers

The y-component of vector (a ) is  -7.0j m/s²

In science, anything that has both a direction and a magnitude is referred to as a vector. They are typically represented by pointing arrows, the length of which denotes the size of the vector. The direction is then crucial in accordance with the vector.

We are given that,

the x -component of vector (a)x = 7.0m/s²

The acceleration in vector sum form x - direction and y - direction is given as,

[tex]a = a_{x} +a_{y}[/tex]

So that , the (-y-direction) acceleration then x- component must be zero then,

[tex]a =(7.0i) + (-7.0j)\\a= - 7.0j[/tex]

Thus, the acceleration on -y- direction will be -7.0jm/s²

To know more about vector

https://brainly.com/question/11238162

#SPJ1

anne is working on a research project that involves the use of a centrifuge. her samples must first experience an acceleration of 100???? , but then, the acceleration must increase by a factor of eleven. by how much will the rotational speed have to increase? express your answer as a fraction of the initial rotation rate.

Answers

if the final acceleration increases by a factor of 11, then the final rotation should be 3.32[tex]w_{1}[/tex] times the initial rotation.

The relation between the centripetal acceleration and rotation speed of the particle is given as

a=rω²    

Where ω  is the angular speed and r radius of the circle

Now, rearranging the above equation we will get

ω = √[tex]\frac{a}{r}[/tex]

The initial acceleration is given by

           a₁ =100g

But then, the acceleration increases by a factor of 11. Which means the final acceleration of the particle is

a₂ = 11 (100g)

    = 1100g

The final and initial rotation speeds are related as

[tex]\frac{w1}{w2}[/tex]   = √[tex]\frac{a2}{a1}[/tex]

[tex]w_{2}[/tex]  = √[tex]\frac{1100g}{100g}[/tex] x [tex]w_{1}[/tex]

[tex]w_{2}[/tex]  = 3.32 [tex]w_{1}[/tex]

The final rotation should be 3.32[tex]w_{1}[/tex] times the initial rotation.

If you need to learn more about acceleration click here:

https://brainly.com/question/25749514

#SPJ4

An arrow is launched straight up from the ground with an initial velocity of 23.4m/s how long until it reaches its highest point

Answers

Answer:

t = 2.388 seconds

Explanation:

Initial velocity is 23.4 m/s

At the highest point, the velocity will be 0 m/s

We can use the equation

[tex]v = u+at[/tex]

V is the final velocity at the highest point

U is the initial velocity

A is the gravity pulling down on the ball

T is the time in seconds  

We plug in the numbers

[tex]0 = 23.4 - 9.8t[/tex]

[tex]t = 2.387755102[/tex]

You are instructed by your supervisor to move 20 boxes—each of which has a weight of 11.34 N—onto an awaiting truck. The back of the truck is 1.65 m above the loading dock. How much work must you perform to do your assignment?

Answers

The amount of work done to move 20 boxes each of weight 11.34 N onto an awaiting truck with back 1.65 m above the loading dock is 374.22 Joule.

Work done is calculated by the product of total force and displacement.

According to the question, total force done to lift the boxes is twenty times the weight of each box. So, the force required to lift all the blocks is,

Force = 20 × 11.34 = 226.8 N

As the back of the truck is 1.65 m above the loading dock, displacement done to move the boxes is 1.65 m.

So, the work done is,

Work = 226.8 × 1.65 = 374.22 Joule

Learn about the elements necessary to accomplish the work at:

https://brainly.com/question/13988204

#SPJ1

A rectangular loop of wire with a cross-sectional area of 2.936 m2 carries a current of 6.488 A. The loop is free to rotate about an axis that is perpendicular to a uniform magnetic field strength of 1.108 T. The plane of the loop is initially at an angle of 59.914o to the direction of the magnetic field. What is the magnitude of the torque on the loop ?

Answers

ANSWER:

10.58 Nm

STEP-BY-STEP EXPLANATION:

Given:

Area (A) = 2.936 m²

Current (i) = 6.488 A

Magnetic field (B) = 1.108 T

Angle (θ)= 59.914°

To calculate the torque we must resort to the following formula:

[tex]\begin{gathered} \tau=MB\sin\alpha \\ \\ \alpha=90\degree-\theta=90-59.914\degree \\ \\ \alpha=30.086\degree \end{gathered}[/tex]

We need to know the magnetic moment, as follows:

[tex]\begin{gathered} M=NIA \\ \\ M=1\cdot6.488\cdot2.936 \\ \\ M=19.05\text{ A}\cdot m^2 \end{gathered}[/tex]

In this way we can calculate the torque:

[tex]\begin{gathered} \tau=19.05\cdot1.108\cdot\sin(30.086\degree) \\ \\ \tau=10.58\text{ Nm} \end{gathered}[/tex]

Therefore, the magnitude of the torque on the loop is 10.58 Nm

A girl is outside walking on level ground. First she walks 829 m to the east, then 223 m north, and finally 387 m west. What is the magnitude of her displacement? Give your answer in meters correct to at least 1 decimal place. Do not include units in your answer.​

Answers

Answer:

not enough information need more information to answer the question

Two electromagnetic waves are traveling through a material. Wave 1 has a maximum electric field strength that is three times the maximum field strength of wave 2. How do the average intensities of the waves compare?.

Answers

Wave 1's intensity is nine times greater than Wave 2's. An electromagnetic wave carries both magnetic and electric energy when it moves.

What is electromagnetic waves?

An electromagnetic wave carries both magnetic and electric energy when it moves. Both waves contribute equally to the energy of electromagnetic waves. It is assumed that wave 1 has an electric field E that is three times greater than wave 2's maximal electric energy E'.

Thus, we can state:

E/E' = 3

We are aware,

The wave's intensity is determined by,

I = 1/2eE²c

Where,

I represents the wave's power,

e is a free space permittivity,

E represents the wave's energy.

The speed of light is c.

the strength of wave 1 right now

I1 = 1/2eE1²c

second wave

I2 = 1/2eE²c

Currently, dividing the two intensities,

I1/I2 = E1²/E2²

We are aware,

E/E' = 3

So,

I1/I2 = 9

I1 = 9I2

Thus, we can state:

Wave 1's intensity is nine times greater than Wave 2's.

To learn more about electromagnetic waves refer to:

https://brainly.com/question/25847009

#SPJ4

On the kickoff, the football was in the air for 4 seconds before hitting the ground 52 meters away. How fast did the ball leave the foot of the kicker?

Answers

The initial speed that the ball left the foot of the kicker if it was in the air for 4 seconds before hitting the ground 52 meters away is 23.64 m / s

T = 2 u sin θ / g

R = u cos θ * T

T = Total time of flight

u = Initial velocity

g = Acceleration due to gravity

R = Horizontal range

T = 4 s

R = 52 m

4 = 2 u sin θ / 9.8

u = 39.2 / 2 sin θ

u = 19.6 / sin θ → ( 1 )

52 = u cos θ * 4

u = 13 / cos θ → ( 2 )

Equating ( 1 ) and  ( 2 ),

19.6 / sin θ = 13 / cos θ

sin θ / cos θ = 19.6 / 13

tan θ = 1.5

θ = 56.3°

From ( 2 )

u = 13 / cos 56.3°

u = 13 / 0.55

u = 23.64 m / s

Therefore, the initial speed that the ball is 23.64 m / s

To know more about Horizontal range of projectile

https://brainly.com/question/23827445

#SPJ1

In the principle of moments (select the correct statement):
20 points
the moment of the biggest Force will be always bigger than the moment of the other force
moments of both forces are equal
the moment of the smallest Force will be always bigger than the momentof the other Force

Answers

According to the research, the correct option is moments of both forces are equal. In the principle of moments, both moments of forces are equal.

What is the principle of moments?

It states that the moment of a force about a point is equal to the sum of the moments of the components of the force about the point where two forces are equivalent when their effects on a rigid body are the same, that is, produce the same moment.

In this sense, two forces acting on an object are known as a couple whose torques about a point cause the object they act on to tend to rotate.

Therefore, we can conclude that following the equilibrium condition in the principle of moments, a structure of an object has to respond to the action of a moment with another of the same value and in the opposite direction.

Learn more about the principle of moments here: https://brainly.com/question/26117248

#SPJ1

a rigid bar with mass m, length l, and a uniformly distributed positive charge q is free to pivot about the origin in the presence of a spatially uniform electric field e⃗

Answers

a. V(y)=V-E(y) b. U=vq-1/2QEL.cosФ c. V=EL/2 d. angular speed of the bar as it passes Ф=0° is W=[tex]\sqrt{3QE/ML}[/tex]

You were already familiar with the ideas of velocity and speed. The idea of angular speed and velocity, however, is what needs to be understood in terms of physical numbers. Angular motion is the term for when an item is expected to travel along a circular path while making a specific angle. The idea of angular speed and angular velocity is derived from the object's angular motion. Let's examine these ideas in greater depth. Assuming you are rotating a ball in a circular orbit, the angular speed can be defined as follows.

A body's rate of angle change over time as it rotates in a circular orbit is known as its angular speed. The definition of angular velocity is similar to that of linear velocity when an object is moving with some speed in a circular orbit.

Learn more about angular speed  here:

https://brainly.com/question/15584705

#SPJ4

replace the force system acting on the post by a resultant force, and specify where its line of action intersects the post ab measured from point b. (10 points)

Answers

The resultant force is the force that can replace all the acting forces and have a comparable effect on the section or beam when several forces are acting on it.

If the forces acting on a body in a specific direction result in a net sum of zero in that direction, the forces acting on the body are said to be in equilibrium.

The resultant force is the force that can replace all other forces operating on a beam or section while still having the same impact on the section or beam. This force is used when many forces are acting on a beam or section.

The resultant force in the X direction equals 250 and must act for more than five, minus 500 cosign 30 minus 300, which equals 533.41 Newton and the magnitude of the forces in the left direction. The magnitude of the resultant force in the y direction should likewise be the same. S

ome forces that are equal to 500 sign 30 - 150 in the y direction. multiply it by 3/5 so that it equals 100. Newton's are inefficient, thus the force Rf's magnitude should be square root off 533.1 square lost 100 square, equaling 542 Newtons, and the angle Sita if 10 members 100 over 533.41, equal to 10.6 degrees for the location off.

We have 533.1 when we multiply the equal 500 committed by the result on force So instant off the resultant around the. 30 supply point to minus 250 and multiplied by minus 500. city above five, apply.

Subtract 200 50 from 0.5. Divide by 4/5. Loss of three supplies A 300 must submit one. Therefore, using this equation, we can determine T's value, which is 827 millimeters.

To know more about force click on the link:

https://brainly.com/question/13191643

#SPJ4

what is temperature?
how temperature can be measured​

Answers

Answer:

Temperature is the kinetic energy of the particles of a substance.

Explanation:

The more kinetic energy a particle has the higher it's temperature. In the case of the atmosphere, which is what we are primarily concerned with in Meteorology, we measure this using a mercury thermometer (in certain situations we use an alcohol thermometer and of course modern times have given us things like dewcells and digital thermometers but we always go back to the mercury thermometer for accuracy).

Hydrochloric acid and sodium hydroxide can be combined to form sodium chloride and water. Which best explains the reaction?

The mass of sodium hydroxide will result in the same mass of sodium chloride.
The total mass of hydrochloric acid and sodium hydroxide will result in a lower total mass of sodium chloride and water.
The mass of hydrochloric acid will result in the same mass of sodium chloride.
The total mass of hydrochloric acid and sodium hydroxide will result in the same total mass of sodium chloride and water.



THE ANSWER IS D

Answers

The statement that best describes the reaction is as follows: The total mass of hydrochloric acid and sodium hydroxide will result in the same total mass of sodium chloride and water (option D).

What is the law of conservation of matter?

The law of conservation of matter states that matter can neither be created nor destroyed. This law means that the mass that enters and leave a system is accounted for.

The law of conservation of mass explains the input and output of mass in a chemical reaction such that the mass of the reactants must be equal to the mass of the products.

According to this question, hydrochloric acid (HCl) and sodium hydroxide (NaOH) can be combined to form sodium chloride and water.

This means that the total amount of matter of the reactants (hydrochloric acid and sodium hydroxide) must equate the total amount of matter of the products (sodium chloride and water).

Learn more about law of conservation of mass at: https://brainly.com/question/28711001

#SPJ1

Your friend states in a report that the time needed for ten laps has been measured and that the average time required to circle the 2.5-mile track was 65.421 seconds. You know that the clock used has a precision of .2 seconds. How much confidence do you have in the results of the report? Explain

Answers

Results are never more exact than the least precise measurement. The average lap time surpasses clock precision.  I will have a fair amount of confidence in the results of the report

This is further explained below.

What is a circle?

Generally, A result can never be more accurate than the measurement that was the least accurate.

The calculated average lap time is more accurate than the clock can measure.

A conclusion can never be more accurate than the measurements that were used to get at it. The accuracy that may be achieved with the clock is exceeded by the average lap time that was determined.

Hence, for a clock that bears a precision of 0.2 seconds, I will be confident in its results bearing precision in mind, Because knowing the precision you can easily manipulate the results to get the desired outcome

Read more about circle

https://brainly.com/question/11833983

#SPJ1

two carts mounted on an air track are moving toward one another. cart 1 has a speed of 0.9 m/s and a mass of 0.45 kg. cart 2 has a mass of 0.62 kg. (a) if the total momentum of the system is to be zero, what is the initial speed of cart 2 in meters per second?

Answers

Because the total momentum of the system is zero, Cart 2's initial speed is 0.62 m/s. No, the system's kinetic energy is not equal to zero because energy, according to the law of conservation of energy, cannot be destroyed.

The kinetic energy of the system is 0.268 Joules. Linear momentum is a vector quantity defined as the product of an object's mass, m, and velocity, v. The letter 'p' is used to express it, and momentum is used for short. Please keep in mind that the body's momentum always points in the same direction as its velocity vector. A body's momentum is defined as the product of its mass and velocity.

Learn more about momentum here-

https://brainly.com/question/24030570

#SPJ4

Other Questions
consider the following class definition. public class examscore { private string studentid; private double score; public examscore(string sid, double s) { studentid 45.6 divided by 0.03 Use I = PTR to solve (time is in years) I=$252R=7% find T.P=$900 why is the percent daily value for protein omitted from the nutrition facts food label? multiple select question. it would be too confusing for the consumer. most people do not know or care about their protein intake. protein is not a public health concern for the majority of people. it would be necessary to know the protein quality of a food which is expensive to determine. lena is scheduling 35 part-time employees to complete a project. of those employees, 1/5 work mornings only, 10 work only afternoons, and the rest alternate between mornings and afternoons. if lena plans on using all of her morning only staff, 1/2 of her afternoon only staff, and 1/3 of her remaining staff, how many people will lena schedule? which psychological theory would support the claim that our sleep habits evolved as a protective measure against predatory risks, which are higher when it is dark out? I am not sure if this is true or false. Please help URGENT!!!!!!!!!Thank you to whoever solves it right! HELP ASAP!!Solve the inequality divided by negative 3. 2 is greater than or equal to 7. 6 for r. A. ) r 24. 32 B. )r 24. 32 C. )r 2. 375 D. ) r 2. 375 Users in motion, wireless connectivity, and a cloud-based resource are the elements in a ________ system. HELP ASAP I NEED ANSWER NOWW IT'S MISSING PLEASE HELP 50 POINTS question 4, select all of the things that happened during the reign of terror In addition to the problems stated here, what other issue contributed to the problems faced by native americans on the great plains at the end of the 19th century?. Martin eats 3/8 of a pizza. Felix eats 2/5of a pizza ofsame size. Who eats more pizza? How much more pizzadoes he eat? The signs for "mother" and "daughter" both involve touching the forehead. A. True B. False The Picture Has The Question. a) Find the value of 8*1/3b) Find the value of 8*2/3c) Find the value of 164*3/4 do you think the effect of the soanish anerican war were benficial or detrimentsl to the united stats ASAP!! 3x + 9x = 6x + 42Solve the equation for X and include all steps for Branliest. Given the decomposition reaction:2SI3(g)->2SO2(g) + O2(g)According to Le Chteliers principle, what will happen when the volume of the container is increased for the chemical reaction that had reached equilibrium? A)Increasing volume, increases pressure and favors the products. B) Increasing volume, decreases pressure and favors the products.C)Increasing volume, increases pressure and favors the reactants.D) Increasing volume, decreases pressure and favors the reactants.